Question and Answers Forum

All Questions      Topic List

Logarithms Questions

Previous in All Question      Next in All Question      

Previous in Logarithms      Next in Logarithms      

Question Number 119802 by bemath last updated on 27/Oct/20

Given a,b,c real number and not equal to 1.  If log _a (b)+log _b (c)+log _c (a)=0 then   (log _a (b))^3 +(log _b (c))^3 +(log _c (a))^3 =?

$${Given}\:{a},{b},{c}\:{real}\:{number}\:{and}\:{not}\:{equal}\:{to}\:\mathrm{1}. \\ $$$${If}\:\mathrm{log}\:_{{a}} \left({b}\right)+\mathrm{log}\:_{{b}} \left({c}\right)+\mathrm{log}\:_{{c}} \left({a}\right)=\mathrm{0}\:{then}\: \\ $$$$\left(\mathrm{log}\:_{{a}} \left({b}\right)\right)^{\mathrm{3}} +\left(\mathrm{log}\:_{{b}} \left({c}\right)\right)^{\mathrm{3}} +\left(\mathrm{log}\:_{{c}} \left({a}\right)\right)^{\mathrm{3}} =? \\ $$

Answered by $@y@m last updated on 27/Oct/20

If l+m+n=0  then l^3 +m^3 +n^3 =3lmn  Here l=log _a (b)  m=log _b (c)  n=log _c (a)  ∴ 3lmn=3×log _a (b)×log _b (c)×log _c (a)=3

$${If}\:{l}+{m}+{n}=\mathrm{0} \\ $$$${then}\:{l}^{\mathrm{3}} +{m}^{\mathrm{3}} +{n}^{\mathrm{3}} =\mathrm{3}{lmn} \\ $$$${Here}\:{l}=\mathrm{log}\:_{{a}} \left({b}\right) \\ $$$${m}=\mathrm{log}\:_{{b}} \left({c}\right) \\ $$$${n}=\mathrm{log}\:_{{c}} \left({a}\right) \\ $$$$\therefore\:\mathrm{3}{lmn}=\mathrm{3}×\mathrm{log}\:_{{a}} \left({b}\right)×\mathrm{log}\:_{{b}} \left({c}\right)×\mathrm{log}\:_{{c}} \left({a}\right)=\mathrm{3} \\ $$

Answered by Ar Brandon last updated on 27/Oct/20

log_a b+log_b c+log_c a=0  ⇒(log_a b+log_b c)^3 =−(log_c a)^3   ⇒(log_a b)^3 +3(log_a b)^2 (log_b c)  +3(log_a b)(log_b c)^2 +(log_b c)^3 =−(log_c a)^3   ⇒(log_c a)^3 +(log_a b)^3 +(log_b c)^3        =−3(log_a b)(log_b c)[log_a b+log_b c]       =3(log_c a)(log_a b)(log_b c)       =3((1/(log_a c)))(((log_a b)/1))(((log_a c)/(log_a b)))=3

$$\mathrm{log}_{\mathrm{a}} \mathrm{b}+\mathrm{log}_{\mathrm{b}} \mathrm{c}+\mathrm{log}_{\mathrm{c}} \mathrm{a}=\mathrm{0} \\ $$$$\Rightarrow\left(\mathrm{log}_{\mathrm{a}} \mathrm{b}+\mathrm{log}_{\mathrm{b}} \mathrm{c}\right)^{\mathrm{3}} =−\left(\mathrm{log}_{\mathrm{c}} \mathrm{a}\right)^{\mathrm{3}} \\ $$$$\Rightarrow\left(\mathrm{log}_{\mathrm{a}} \mathrm{b}\right)^{\mathrm{3}} +\mathrm{3}\left(\mathrm{log}_{\mathrm{a}} \mathrm{b}\right)^{\mathrm{2}} \left(\mathrm{log}_{\mathrm{b}} \mathrm{c}\right) \\ $$$$+\mathrm{3}\left(\mathrm{log}_{\mathrm{a}} \mathrm{b}\right)\left(\mathrm{log}_{\mathrm{b}} \mathrm{c}\right)^{\mathrm{2}} +\left(\mathrm{log}_{\mathrm{b}} \mathrm{c}\right)^{\mathrm{3}} =−\left(\mathrm{log}_{\mathrm{c}} \mathrm{a}\right)^{\mathrm{3}} \\ $$$$\Rightarrow\left(\mathrm{log}_{\mathrm{c}} \mathrm{a}\right)^{\mathrm{3}} +\left(\mathrm{log}_{\mathrm{a}} \mathrm{b}\right)^{\mathrm{3}} +\left(\mathrm{log}_{\mathrm{b}} \mathrm{c}\right)^{\mathrm{3}} \\ $$$$\:\:\:\:\:=−\mathrm{3}\left(\mathrm{log}_{\mathrm{a}} \mathrm{b}\right)\left(\mathrm{log}_{\mathrm{b}} \mathrm{c}\right)\left[\mathrm{log}_{\mathrm{a}} \mathrm{b}+\mathrm{log}_{\mathrm{b}} \mathrm{c}\right] \\ $$$$\:\:\:\:\:=\mathrm{3}\left(\mathrm{log}_{\mathrm{c}} \mathrm{a}\right)\left(\mathrm{log}_{\mathrm{a}} \mathrm{b}\right)\left(\mathrm{log}_{\mathrm{b}} \mathrm{c}\right) \\ $$$$\:\:\:\:\:=\mathrm{3}\left(\frac{\mathrm{1}}{\mathrm{log}_{\mathrm{a}} \mathrm{c}}\right)\left(\frac{\mathrm{log}_{\mathrm{a}} \mathrm{b}}{\mathrm{1}}\right)\left(\frac{\mathrm{log}_{\mathrm{a}} \mathrm{c}}{\mathrm{log}_{\mathrm{a}} \mathrm{b}}\right)=\mathrm{3} \\ $$

Terms of Service

Privacy Policy

Contact: info@tinkutara.com